Análise real/Convergência pontual: diferenças entre revisões

[edição não verificada][edição não verificada]
Conteúdo apagado Conteúdo adicionado
Sem resumo de edição
Sem resumo de edição
Linha 55:
\end{array}
\right.</math>
 
== Exercícios ==
;Problema 1
Considere a seguinte seqüência de funções <math>f_n:\mathbb{R}^*_+\to\mathbb{R}\,</math> dada pela relação de recorrência:
:<math>\begin{array}{l}
f_1(x)=x\\
f_n(x)= \frac{1}{2}\left[f(x)+\frac{x}{f(x)}\right]\,
\end{array}</math>
Mostre que:
 
*<math>f_n(x)\to \sqrt{x}</math> pontualmente
*<math>f_n(x)\to \sqrt{x}</math> uniformemente em cada intervalo <math>[a,b]\,</math> contanto que <math>0<a<b\,</math>.
*a convergência não é uniforme em nenhum intervalo do tipo <math>(0,a)\,</math> nem do tipo <math>(a,\infty)\,</math> com <math>a>0\,</math>.
 
 
;Problema 2
Considere a seqüência de funções indexada pelos índices <math>n\,</math> e <math>m\,</math>:
* <math>f_{mn}(x)= \cos^{2n}(m!\pi x) ~x\in\mathbb{R}, ~~ n=1,2,3,\ldots\,</math>
Mostre que:
* <math>\lim_{m\to\infty}\lim_{n\to\infty} f_{mn}(x)=\left\{
\begin{array}{rl}
1,&x \in \mathbb{Q}\\
0,&\hbox{c.c.}
\end{array}
\right.</math>
 
 
;Problema 3
Considere a seqüência de funções <math>f_n:[0,1]\to\mathbb{R}\,</math> definidas por:
*<math>f_n=\left\{
\begin{array}{ll}
n^2x,&x\in [0,1/n]\\
2n- n^2 x,&x\in (1/n,2/n)\\
0,&x\in [2/n,1]
\end{array}
\right.</math>
Mostre que
*<math>\lim_{n\to\infty} f_n(x) =0\,</math>
não obstante
*<math>\lim_{n\to\infty}\int_{0}^1 f_n(x)dx =1/2\,</math>
 
 
;Problema 4
Defina <math>f_n:\mathbb{R}\to\mathbb{R}\,</math> como:
*<math>f_n=\left\{
\begin{array}{ll}
1/n,&x\in |x|<n\\
0,&x\in |x|\geq n\\
\end{array}
\right.</math>
 
Mostre que
*<math>\lim_{n\to\infty} f_n(x) =0\,</math> uniformemente
não obstante
*<math>\lim_{n\to\infty}\int_{0}^1 f_n(x)dx =2\,</math>
 
 
;Problema 5
Seja a seqüência de funções <math>f_n:[0,\infty)\to\mathbb{R}\,</math> dada por:
* <math>f_n(x)=\left\{
\begin{array}{ll}
\left(1-\frac{x}{n}\right)^n,& x\in [0,n]\\
0, &x\in (n,\infty)
\end{array}
\right.</math>
Mostre que:
* <math>0\leq f_n(x) \leq e^{-x}\,</math>
* <math>f_n(x) \to e^{-x}\,</math> uniformemente em <math>[0,M]\,</math> para cada <math>M>0\,</math>
Conclua, provando que:
* <math>\lim_{n\to\infty}\int_0^n \left(1-\frac{x}{n}\right)^n = 1\,</math>
 
;Problema 6
Construa uma seqüência de funções contínuas em <math>[0,1]\,</math> convergindo pontualmente para um função que não é integrável à Riemann.
 
==Ver também ==